phd nursing meaning

Home / Getting Your Ph.D. in Nursing

Getting Your Ph.D. in Nursing

Becoming a ph.d. nurse, what does a ph.d. nurse do, ph.d. nurse salary & employment, ph.d. vs. dnp in nursing, helpful organizations, societies, and agencies.

Getting Your Ph.D. in Nursing

What Is a Ph.D. Nurse?

A Ph.D. nurse is one who has completed a Doctor of Philosophy in Nursing degree. A Ph.D., or doctoral degree, is the highest level of education a nurse can achieve. Different from a Doctor of Nursing Practice (DNP) degree, which focuses on advanced clinical practice, a Ph.D. in Nursing program is a research and science-focused degree that prepares nurses for careers conducting important medical research that will advance the entire nursing profession and for teaching nursing at the college level.

In order to become a Ph.D. nurse, of course, nurses must complete a Ph.D. in nursing program, which generally takes 4 to 6 years to finish. An aspiring Ph.D. nurse must have a strong interest in conducting medical research and/or teaching future nurses. Strong leadership skills are also important, as many Ph.D. nurses go on to supervise and mentor other nurses, whether they work in scientific research, management, or teaching capacity.

What Are the Educational Requirements For a Ph.D. Nurse Program?

A Ph.D. in Nursing program is known as a terminal degree, meaning it is the highest level of education for the nursing profession (in addition to the DNP degree, another separate nursing doctorate program track). Prior to entering a Ph.D. program, nurses must complete a Bachelor of Science in Nursing (BSN) degree and pass the NCLEX-RN exam. In some cases, applicants to a Ph.D. in Nursing program must also complete a Master of Science in Nursing (MSN) degree, which provides advanced education in nursing practice with courses in pharmacology, pathophysiology, and clinical practice.

Educational Prerequisites

Specific requirements to complete a Ph.D. in Nursing program will vary slightly from school to school. Schools offer Ph.D. in Nursing programs in traditional classroom formats, as well as online and hybrid styles that combine in-person study with online coursework. In addition to a variety of formats for Ph.D. in Nursing programs, students can also sometimes opt to take these programs on a full-time or part-time basis to suit their personal schedules.

The curriculum for a Ph.D. in Nursing program is research-focused, with coursework in advanced scientific research principles, data analysis, and statistical measurement. Ph.D. programs generally culminate in a dissertation and original research project. As an example of Ph.D. curriculum, below is a selection of courses offered by the Medical University of South Carolina as part of their online Ph.D. in Nursing Science program:

  • Advanced Quantitative Research Methods
  • Qualitative Research Methods
  • Advanced Statistical Methods for Nursing Research
  • Advanced Study Design and Methods
  • Advanced Health Policy & Advocacy
  • Research Team Leadership

A Ph.D. nurse conducts scientific research that advances the nursing profession. The knowledge that Ph.D. nurses gather and present as a part of their scientific research powers positive change in the quality of patient care and outcomes in the entire nursing field. In addition to their role as nurse scientists, Ph.D. nurses also teach and mentor nurses at the college/university level, working to shape the next generation of nurses.

What Are the Roles and Duties of a Ph.D. Nurse?

The majority of Ph.D. nurses pursue careers in either the research or teaching fields, so their day-to-day duties will vary depending on which career track they have chosen.

For a nurse researcher , typical duties may include:

  • Identify research questions, and design and conduct scientific research in partnership with other scientists from various fields
  • Collect and analyze scientific data and publish reports detailing findings
  • Write proposals and apply for grants to help fund their research
  • Establish and maintain quality assurance programs to ensure the validity of their data findings
  • Train and supervise laboratory staff and other nurses or scientists

For a Ph.D.  nurse educator who has chosen to pursue a faculty position, typical duties may include:

  • Plan, prepare, and revise curriculum and study materials for nursing courses
  • Deliver lectures to undergraduate and graduate level nursing students
  • Supervise students' laboratory and clinical work
  • Grade students' classwork, laboratory, and clinical performance
  • Mentor and advise students regarding their future work in the nursing industry

For faculty members who pursue department chair or administration roles, additional duties may include:

  • Hire, supervise and conduct performance reviews of faculty members
  • Assist with the scheduling of classes and professors
  • Oversee department curriculum and provide quality control as to the content and materials of given nursing courses

Workplace Settings

A Ph.D. nurse can work in a variety of settings, depending on the career path he or she has chosen. A Ph.D. nurse may find employment at a hospital, medical laboratory, research facility, or university as a research scientist, or may work at a nursing school, college, or university as a faculty member or department chair. In some cases, a Ph.D. nurse may also work as a public health nurse in a government setting, helping to develop research-based solutions to public health issues.

Salaries for Ph.D. nurses vary based on the type of employment a nurse seeks after graduation. Nurse researchers, a primary career path for Ph.D. nurses, can expect a median salary of $90,000 according to Payscale.com. For Ph.D. nurses who pursue a teaching position, the median annual wage for post-secondary nursing instructors is $77,440 according to the U.S. Bureau of Labor Statistics as of May 2021. Geographical location, career length, and experience level are all factors that can influence a Ph.D. nurse's annual salary.

The nursing profession as a whole has a particularly bright employment outlook, with the employment of registered nurses projected to grow 9 percent from 2020 to 2030 according to the BLS. In addition, a large number of healthcare facilities are looking for nursing professionals with higher degrees, which means the demand for Ph.D. and DNP level nurses will continue to grow. In fact, the Institute of Medicine 's 2015 "The Future of Nursing Report" emphasized the need for more Ph.D. level nurses.

As there are two doctorate-level nursing program types to choose from, there may be some confusion as to the differences between a Ph.D. nursing program and a Doctor of Nursing Practice (DNP) degree. The primary difference between the two programs relates to nurses' career aspirations. A DNP program trains nurses to perform the highest level of nursing practice and to translate research into high-quality patient care, while a Ph.D. program prepares nurses to conduct cutting-edge research that will advance the science of nursing and patient care. In addition to research positions, a Ph.D. program prepares nurses for leadership and teaching positions at hospitals and colleges/universities. To simplify, a DNP is a nursing practice doctorate degree, while a Ph.D. is a research and teaching doctorate.

Other key differences between DNP and Ph.D. programs are curriculum and program length. A typical DNP program includes courses on advanced nursing practice, leadership, and management topics and requires patient care clinical hours as well as a final capstone project. A Ph.D. program includes coursework on research methodologies, data analysis, and healthcare leadership and policy, and requires students to complete original research and a dissertation. In general, a Ph.D. program takes longer to complete than a DNP program, with Ph.D. programs taking an average of 4 to 6 years to complete and a DNP program taking 3 to 4 years, but can be completed in as little as 2 years depending on the school and program chosen.

  • American Association of Colleges of Nursing
  • American Nurses Association
  • International Association of Clinical Research Nurses
  • National Institute of Nursing Research

Nurse.org

DNP vs PhD in Nursing - What is the Difference?

  • What's the Difference Between a DNP vs a PhD?

DNP vs PhD Compared

Deep dive into dnp vs phd.

Doctor smiling while looking over charts and notes

What's the Difference Between a DNP vs a PhD?

Nurses who have earned a master’s degree and are ready for the next step may be choosing between a DNP vs PhD. Both are considered terminal degrees for advanced practice nurses but are rooted in very different practices. In the most general terms, a  doctor of nursing practice (DNP) is a clinical practice degree, while a Doctor of Philosophy in Nursing (PhD) is a research-focused degree. 

Nurse Practitioners have the option to earn either degree, but a DNP is preferred. Individuals with a Master’s degree in Nursing Education will generally earn their PhD.

Popular Online Doctor of Nursing Practice (DNP) Programs

Grand Canyon University

GCU's College of Nursing and Health Care Professions has a nearly 35-year tradition of preparing students to fill evolving healthcare roles as highly qualified professionals. GCU offers a full spectrum of nursing degrees, from a pre-licensure BSN degree to a Doctor of Nursing Practice (DNP) program.

Enrollment: Nationwide

  • DNP - Educational Leadership

Georgetown University

Enrollment: Nationwide, excluding NY and WA.

  • DNP - Family NP
  • DNP - Adult Gerontology Acute Care NP
  • DNP - Nurse-Midwifery/Women's Health NP
  • DNP - Women's Health NP

Grand Canyon University

  • DNP - Psychiatric Mental Health (Post-MSN)
  • DNP - Psychiatric Mental Health NP (Post-BSN)

Grand Canyon University

RNs with an associate degree: Advance your nursing career with a supportive online RN to BSN program for women. Simmons University has been empowering nursing leaders for more than 100 years and offering innovative online degree programs since 2013. Up to 96 transfer credits accepted. Request information now.

Grand Canyon University

Program Length

  • BSN to PhD: 5-10 years
  • MSN to PhD: 3-7 years
  • ADN-to-DNP: 5-6 years
  • BSN-to-DNP: 3-4 years
  • MSN-to-DNP: 2 years

Average Salary

  • PhD nurse educators had a median annual wage of $78,580, according to the U.S. Bureau of Labor Statistics as of May 2018.
  • According to Payscale,  nurse researchers earned an average salary of $81,500.
  • Healthcare salary trends suspect DNPs average $94,500to $134,500 per year. This ultimately depends on the place of employment. This ultimately depends on the place of employment.

Job Opportunities

  • The greatest need for PhD prepared nurses is in academia. According to 2018, Special Survey on Vacant Faculty Positions 1,715 faculty vacancies was identified. There is also a need for 138 additional positions to fit the current demand. 
  • According to the AACN, the biggest opportunity for DNP individuals is also in academia. The data show a national nurse faculty vacancy rate of 7.9%. Most of the vacancies (90.7%) were faculty positions requiring or preferring a doctoral degree.

Certification

  • Renewal: None
  • Renewal: 5 years. Retake the exam or provide proof of at least 1,000 clinical practice hours and 75 CE unit

Work Environments

  • Colleges & Universities
  • Research facility
  • Medical laboratory
  • Government agency
  • Universities
  • Public Health Offices
  • Specialty Practices
  • Autonomous Practices
  • Health Care Administration Settings
  • Health Care Policy Advocacy Organizations

Career Options after Graduation

  • Nursing researcher
  • Health policy positions
  • Nursing faculty positions
  • Leadership in nursing practice
  • Management positions
  • Healthcare policy, administration, or government positions
  • Academia in practice-based nursing programs

What is a DNP?

Doctors of Nursing Practice (DNP) deliver high-quality advanced nursing care in a clinical setting, similarly to an NP; however, these individuals have taken their careers a step further.

The American Association of Colleges of Nursing (AACN) states that transitioning from advanced practice NP degrees to the doctoral level is a “…response to changes in health care delivery and emerging healthcare needs, and additional knowledge or content areas have been identified by practicing nurses. In addition, the knowledge required to provide leadership in the discipline of nursing is so complex and rapidly changing that additional or doctoral level education is needed.”

Essentially, DNP graduates are leaders in advanced nursing practice who bring evidence-based knowledge into the clinical setting to help improve healthcare outcomes and strengthen the leadership role of nurses in both clinical and academic settings. 

Want to know more? Check out our guide to learn everything you need to know about the DNP meaning .

The benefits of a DNP program, according to the AACN, are:

  • enhanced knowledge to improve nursing practice and patient outcomes
  • enhanced leadership skills 
  • increased supply of faculty for clinical instruction
  • development of needed advanced competencies for increasingly complex clinical, faculty, and leadership role

>> Show Me DNP Programs

DNP Scope of Practice

An individual with a DNP can function in a provider capacity but most also work to generate new scientific and clinical knowledge in nursing and healthcare. A nurse practitioner with a doctorate does not change the scope of practice. DNPs that work in the clinical setting can, 

  • Order, perform and interpret diagnostic tests
  • Diagnose and treat acute and chronic conditions
  • Record and examine medical history, diagnoses, and symptoms
  • Prescribe medications
  • Manage patients’ overall care
  • Counsel patients and their families
  • Educate patients and families on disease prevention and plan of care
  • Monitor and operate medical equipment
  • Perform physical examinations and patient observations
  • Collaborate with other healthcare professionals
  • Detect changes in a patient’s health and change the treatment plan if necessary

DNPs that work solely in the clinical setting will rarely conduct scientific research or teach. For this reason, most DNPs also work in academia, as administrators, and/or researchers. 

The duties of a nurse with a DNP, outside of the clinical setting, can include the following:

  • Use informatics to lead improvements in healthcare outcomes.
  • Develop or update policies and procedures.
  • Educate patients and public about healthcare.
  • Innovate new healthcare strategies to improve patient health outcomes.

In 23 states, DNPs have “full practice authority” which means they do not have to work under the supervision of a doctor. Full practice states include,

  • North Dakota

In states with reduced practice (Alabama, Wisconsin, and New York) and restricted practice (Virgina, Massachusetts, and California), DNPs must have a medical doctor sign certain medical patient care decisions. DNPs have prescriptive privileges in all 50 states and can administer controlled substances in 49 states.

Map of practice authority for nurse practitioners by state

BLS's May 2022 data shows NPs made an annual median salary of $126,260; interestingly, there is minimal concrete data on the average annual salary of a DNP.   Healthcare salary trends suspect DNPs average $94,500 to $134,500 per year. This ultimately depends on the place of employment.

The U.S. Bureau of Labor Statistics has very limited data on DNP salaries, but the reported average is $135,830. This average is based on only a small number of reported salaries. The key determining factor for DNP salary is the career nurses pursue after obtaining an advanced degree. A Chief Nursing Officer will earn more than a Nursing Professor. Ultimately, additional advanced education is reflective of higher annual salary earnings.

It is impossible to determine which DNP specialty has the highest salary but hospital administrators, DNP-prepared certified nurse midwives, and certified nurse anesthetists are at the top.

DNP Programs, Length of Time, & Classes

According to the 2023 U.S. News & World Report , the top five DNP programs in the country are,   

  • Johns Hopkins University
  • Columbia University, Duke University and Rush University, and University of Washington (TIE)

Johns Hopkins, Duke, and Rush have consistently been in the top 5 DNP schools the last several years. Rush University has the largest DNP program with over 765 enrolled students, while Johns Hopkins has 624 students enrolled, and Duke has 337 DNP students.

DNP programs can be completed both online and in class. To pursue a DNP, individuals already would have completed a traditional or accelerated BSN program and have an MSN. There are only a handful of programs that accept students into the DNP program without an MSN. 

A DNP program completion can take roughly one to four years. This will depend on the program and whether it is being completed on a full-time or part-time basis. Programs are typically between 30-40 credit hours and 1,000 clinical hours. A percentage of clinical hours earned during an MSN program can transfer in some programs. 

Individuals should expect to take the following classes:

  • Advance Leadership
  • Advance Healthcare Policy
  • Clinical Information Systems
  • Evidence Appraisal
  • Project Development
  • Clinical Reasoning
  • Clinical Pharmacology
  • Advanced Health Assessment and Measurement
  • Health Promotion and Risk Reduction Across the Lifespan
  • Epidemiology
  • Ethics for advanced nursing practice

DNP Admission Requirements

Requirements for DNP programs will vary but most will require the following:

  • MSN degree from a regionally accredited higher education institution and a nationally accredited school of nursing
  • GPA of at least 3.0 or higher in the Master’s program
  • Current, unencumbered nursing license
  • RN experience 
  • Letters of Recommendation (both academic and professional references)
  • Official Transcripts (from all previous colleges/universities)
  • Current Resume/CV
  • Goal statement
  • Personal essay
  • Advanced Practice Registered Nursing license
  • Interview with faculty (if moved forward by admissions committee)
  • Test of English as a Foreign Language (TOEFL) or the International English Language Testing System (IELTS) if applicable 
  • Application fee

What is a Doctor of Philosophy in Nursing (PhD)?

A Doctor of Philosophy in Nursing (PhD in Nursing), is a research-focused doctorate in which students conduct research to advance the science and practice of nursing. PhDs are considered the gold standard for terminal degrees in nursing and at one time was the only doctorate option. 

Individuals must have a strong desire to conduct research or teach students as this is generally what this degree is used for. 

PhD Scope of Practice 

The scope of practice for a nurse with a PhD is tricky. Individuals can work as a bedside nurse, but unless they are a CRNP, they do not have any advance responsibilities or roles beyond a normal bedside nurse. 

Nurses who earn a PhD typically fall into two employment categories: nurse researcher and nurse educator. 

For a nurse researcher, typical duties may include:

  • Identify research questions, and design and conduct scientific research
  • Collect and analyze scientific data and publish reports detailing findings
  • Write proposals and apply for grants to help fund their research
  • Establish and maintain quality assurance programs to ensure the validity of their data findings
  • Writing articles and research reports in nursing or medical professional journals or other publications
  • Presenting findings at conferences, meetings, and other professional speaking engagements
  • Train and supervise laboratory staff and other nurses or scientists

For a nurse educator who has chosen to pursue a faculty position at a university or college, typical duties may include:

  • Plan, prepare and revise curriculum and study materials for nursing courses
  • Deliver lectures to undergraduate and graduate-level nursing students
  • Design curricula and develop programs of study
  • Evaluate program outcomes
  • Research and publication of scholarly work
  • Participating in professional associations
  • Speaking engagements at a nursing conference
  • Writing grant proposals
  • Writing or reviewing textbooks and other educational material
  • Seek continuous improvement in the nurse educator role
  • Advise students and perform many of the tasks associated with academic careers
  • Grade students’ classwork, laboratory and clinical performance
  • Mentor and advise students’ regarding their future work in the nursing industry

PhD Programs, Length of Time, & Classes

A PhD program takes a minimum of three years if taking classes full time. Most individuals complete the programs in five to seven years. The curriculum is between 45 and 70 credit hours. There is no direct patient care practicum but there is a dissertation. 

A dissertation is a requirement of all PhD programs, and individuals must defend their manuscript during a presentation at the end of their program. The dissertation is a technical paper that is intended to show the growth of the student throughout the program. It includes theory, research, and experimentation.

>> Related: What Are the Best Nursing Research Topics?

  • Philosophical Perspectives in Health
  • Scientific Perspectives
  • Quantitative Research Design and Methods
  • Qualitative Research Design and Methods
  • Mixed Methods Research Design
  • Grant Writing
  • Measurement in Health Care Research
  • Responsibilities and Activities of the Nurse Scientist 
  • Leadership in Science: The Role of the Nurse Scientist
  • Statistical Methods
  • Dissertation
  • Longitudinal Methods
  • Integrated Interdisciplinary Research Practicum
  • Intervention Research Methods in Health Care

PhD Admission Requirements

Requirements for PhD programs will vary but most will require the following:

  • Graduate of an accredited Master’s in Nursing Program 
  • A written statement of research goals 
  • Research interests that match faculty expertise and School resources
  • GRE scores are accepted but not required
  • Writing sample (publication or graded paper)
  • Resume or curriculum vitae
  • Letters of recommendation (both academic and professional references)
  • Copy of official RN license(s) 
  • Nursing experience
  • Official Transcripts (from all post-secondary schools)

Find Nursing Programs

  • A DNP is a terminal nursing degree rooted in clinical practice and intended for advanced practice nurse practitioners.

What is a PhD?

  • A PhD is a terminal nursing degree rooted in research and intended for individuals wishing to work as nurse educators or nurse researchers. 

What is the difference between DNP vs PhD salary? 

  • Healthcare salary trends suspect DNPs average $94,500to $134,500 per year. This ultimately depends on the place of employment.
  • PhD nurse educators had a median annual wage of $80,780.

Does it take longer to become a DNP or PhD? 

  • It takes about twice as long to earn a PhD DNPs can be earned in as little as one year or full-time study while a PhD takes a minimum of three years.

How long does certification last?

  • DNP’s must renew their certification every five years either through examination or practice hours and continuing education hours. PhD prepared nurses do not have a certification to renew other than their advanced practice degree. 

Are DNPs and PhDs in demand?

  • With an ongoing nursing shortage, an aging population, and retirement of a large percentage of nurse educators, nurses with terminal degrees are in high demand.

Kathleen Gaines

Kathleen Gaines (nee Colduvell) is a nationally published writer turned Pediatric ICU nurse from Philadelphia with over 13 years of ICU experience. She has an extensive ICU background having formerly worked in the CICU and NICU at several major hospitals in the Philadelphia region. After earning her MSN in Education from Loyola University of New Orleans, she currently also teaches for several prominent Universities making sure the next generation is ready for the bedside. As a certified breastfeeding counselor and trauma certified nurse, she is always ready for the next nursing challenge.

Nurses making heats with their hands

Plus, get exclusive access to discounts for nurses, stay informed on the latest nurse news, and learn how to take the next steps in your career.

By clicking “Join Now”, you agree to receive email newsletters and special offers from Nurse.org. You may unsubscribe at any time by using the unsubscribe link, found at the bottom of every email.

Skip to content

Our Culture

Diversity, equity, and inclusion.

Learn about our commitment to social justice and health equality and anti-racism.

Academic Programs

Admissions at a glance.

Learn more about Admissions at Columbia Nursing, including important dates and deadlines, and how to apply to all of our programs. 

Research Centers and Programs

Research areas of focus.

Explore the research areas of focus conducted by our faculty, postdocs, and students.

Patient Care

Primary care services.

The ColumbiaDoctors Primary Care Nurse Practitioner Group, combines evidence-based practice with a personalized approach to provide quality care.

Global Health

Global opportunities for students.

Global opportunities for clinical practicum and research may be available for MDE and doctoral students at Columbia Nursing.

Doctor of Philosophy

Doctor of philosophy (phd), ignite your future with a columbia nursing phd.

The Columbia University School of Nursing PhD program is a full-time, research-intensive curriculum that prepares nurses for careers as nurse scientists who will conduct research across a broad range of populations and health conditions. Importantly, much of our research is focused on health disparity populations with the long-term goal of informing health policy and clinical practice across the lifespan.

Columbia Nursing provides three years (eight semesters) of funding for tuition, related fees, health insurance, and a stipend for full-time PhD students.

Program Design

Our PhD program provides students with an understanding of the philosophical and theoretical underpinnings of nursing science and a strong foundation in research methods (design, statistics, measurement, quantitative and qualitative methods) for clinical, translational and health services research. All students are mentored by research advisors with active programs of research as they move toward independent research and assume the roles of doctorally prepared nurse scientists.  

As a Columbia Nursing PhD student, you will learn to:

  • Design, conduct, and report multidisciplinary research studies that increase knowledge to improve the health and well-being of patients and families across the lifespan
  • Advance the state of the science in a substantive area of research through application of innovative and rigorous methods
  • Promote health and well-being for individuals and families in the context of their communities
  • Provide leadership in improving the health care delivery system at local, national, and international levels
  • Collaborate with other professionals to evaluate and develop policies for delivery of health service
  • Translate evidence accumulated through research into practice and policy at multiple levels

As part of Columbia University Irving Medical Center (CUIMC), Columbia Nursing enjoys a unique collaboration with the College of Physicians and Surgeons, the Mailman School of Public Health, and the College of Dental Medicine. CUIMC provides myriad opportunities for interprofessional collaboration in research . 

The PhD curriculum builds on the foundation of nursing science by bringing together practice, policy, translational research, and leadership. The core courses provide the knowledge and skills necessary to conduct relevant and well-designed research studies. Electives strengthen an area of clinical interest or intensify understanding of a specific research or analytic method. 

Both post-master's and post-BSN students admitted to the program will complete a minimum of 57 credits. The curriculum plan is designed to make it possible to complete the program in three years for those students with clearly defined plans for their dissertation research.

PhD courses are offered in three major areas:

  • Theoretical foundations of nursing science
  • Analytical foundations of nursing science
  • Electives and application

Students must be registered as full-time for the duration of the program (typically three to four years). The minimum number of semester credits in required coursework is 37 (four semesters) for eligibility to progress to the qualifying exam. Six of the 37 credits required to be completed prior to the qualifying examination are elective courses tailored to the student’s dissertation topic and/or dissertation methods. The PhD program requires nine credits of elective courses. A minimum of 57 total credits is required for program completion. 

Concurrent with Coursework

  • Research Experience (participating in faculty research projects and/or a research practicum)
  • Research Faculty Training

Request a Sample Academic Program Plan

Qualifying Examination

The qualifying examination helped me to combine the content I learned in my courses and my research interests so I could further articulate my research question. Performing a scoping review on my topic of interest immersed me in the current literature and was crucial to the development of my dissertation. This experience prepared me to successfully work independently through the rest of my Ph.D.

 Kylie Dougherty, BSN, RN, M.Phil.

In addition to coursework, students must successfully complete a qualifying examination with written and oral components. The Master of Philosophy (MPhil) is awarded after successful completion of the qualifying examination and the student enters doctoral candidacy status.

Dissertation

Students are expected to successfully defend a dissertation reporting original research. Four dissertation credits are required each semester during the dissertation phase of study. 

Scholarship Expectations

My advisor and the Columbia Nursing faculty provided me exceptional guidance throughout the PhD program to extend my learning beyond the classroom with the goal of becoming an independent nurse scientist. I learned valuable skills and knowledge to successfully obtain a NIH-funded predoctoral training award, present research findings at local, regional, and national conferences, and publish manuscripts in peer-reviewed journals with good impact factors.

Joseph Belloir, MSN, RN, PMHNP-BC

  • Publication: At least one manuscript published in an appropriate peer-reviewed journal.
  • Grantsmanship: At least one grant application submitted to an appropriate funding agency or organization.
  • Presentation: At least one abstract submitted for presentation as a poster or oral presentation at an appropriate professional meeting.
  • Networking: Student will attend at least one regional or national research meeting.

Preparation for Postdoctoral Fellowship: Research Career Next Step 

The coursework and research mentoring at Columbia Nursing helped prepare me for the next steps in my education and career post-PhD. In addition to structured coursework and educational seminars, the school provided beneficial informal support and resources. Feedback sessions with both peers and faculty were very helpful in preparing me to present posters and presentations at research conferences. The school also provided funds for travel to conferences where I presented my research. The grant writing workshop and mock reviews of grant applications provided me with tools and feedback needed to successfully apply for additional funding for my research. Finally, interdisciplinary research collaborations with faculty provided me with opportunities to work with researchers from several disciplines to complete my dissertation.

Melissa Beauchemin, PhD '19, MS '10, RN

PhD Student Handbook

The Columbia Nursing PhD student handbook provides information to aid doctoral students in planning coursework and proceeding through all phases of the program.

Request a PhD Student Handbook

What is it like to be a PhD student at Columbia Nursing?

Required courses (excluding electives).

Building upon the foundations provided in the quantitative and qualitative research method courses, in this course students examine advanced methods and frameworks frequently used in studying health policy, health services research problems and comparative effectiveness research. In addition to a critical review of the methods, the course examines the relationship among science, policy and healthcare delivery, and identifies critical questions shaping the future policy research agenda.

Interdisciplinary research is an approach to advancing scientific knowledge in which researchers from different disciplines work at the borders of those disciplines in order to address complex questions and problems. Successful interdisciplinary efforts require mastery of specific competencies. This seminar will introduce students to competencies in interdisciplinary research through a combination of readings, case studies, and lectures in each necessary aspect, chosen from fields essential to successful interdisciplinary research. It is intended to assist learners to understand why and how different professional disciplines must work together to generate and disseminate knowledge. We will examine: different conceptualizations of interdisciplinary; barriers to and facilitators of interdisciplinary research; approaches, benefits, and limitations of collaboration and team science; methods for measuring interdisciplinary collaboration; the intersection of translational and interdisciplinary scientific strategies; and individual researchers' experiences with and evaluations of their own interdisciplinary scientific projects. Learners will develop a set of skills to be effective members and leaders of interdisciplinary research teams.

The student works with a faculty member or other scientist who is conducting a research project. The specific nature of the experience depends on the nature and stage of the research, but might include search and review of relevant literature, data collection, data analysis and/or grant preparation.

This course is intended for PhD students who are engaged in relevant scholarly activities that are associated with dissertation research.

This foundational course will examine the philosophy of nursing knowledge including foundations of nursing theory, concept development, and its application to research. Students will explore approaches to the analysis and development of concepts and the application of nursing concepts and frameworks to clinical practice and research. Ideas, assumptions, events, people, and writings are examined for their influence, inter-relationships, and significance to nursing. Types of reasoning will be evaluated within the context of nursing and health. Major theories, frameworks, and concepts of nursing and health and their implication for research will be discussed. The focus of the course will be on development of critical thinking skills in analyzing key elements of philosophies, concepts, and conceptual frameworks.

In this foundational course students will study the links between theory and the psychosocial and biophysical measures used in nursing research.  Students will employ the principles of classical test theory and item response theory to evaluate the reliability and validity of measurement.  Application of computational techniques will be covered in the lab portion of the course.  Course topics include types and uses of measures, item/scale development and validation, survey methods, reporting for publication, and the relationships between measurement and research ethics, cultural competency, and health disparities.

This course provides a foundation for quantitative research methods and design. Research process topics examined include: appraisal of the quality of existing evidence; identification of gaps in the literature; formulation of researchable questions and testable hypotheses; types of research variables; sampling designs and power analyses; and the uses, strengths, and weaknesses of various experimental and quasi-experimental research designs.

This course provides an in-depth examination of qualitative study designs and methods through a combination of theoretical discussion and hands-on practical experience. Topics include paradigm distinctions, theoretical perspectives, designs and methods, critique of research reports, and ethical issues in qualitative research.

The course is intended for PhD students who are engaged in relevant scholarly activities that are not associated with the required course sequence. Such activities must accrue more than 20 hours/week.

This course is intended to provide a hands-on introduction to delivering data visualizations to serve as a critical lens through which individual and population level health can be examined. The proposed course will combine concepts and theory in data visualization and exploration and practice to enable the student to gain the necessary knowledge to use graphics and statistics to explore the data, find and construct a narrative, and share findings in ways colleagues and decision-makes can readily understand and act upon.

This course is designed to provide the tools for the doctorally prepared nurse to evaluate, translate and integrate published research results into clinical practice. During the course, students will learn how to conceptualize clinical practice problems and transform them into answerable clinical research questions, how to search for the best clinical evidence, and how to assess clinical evidence using basic epidemiological, biostatistical and scientific principles. The course will culminate in a systematic review or meta-analysis of a body of research relevant to advanced practice nursing.

Total Credits:

DNP VS. Ph.D. - 12 Key Differences Between DNP And Ph.D. In Nursing

phd nursing meaning

Nursing is one of the few professions that’s associated with two different terminal degrees: the Doctor of Nursing Practice (DNP) and the Doctor of Philosophy in Nursing (Ph.D.). The former degree is a clinical doctorate designed for advanced practice nurses who are actively working to improve healthcare outcomes in medical settings and who are championing the leadership role that nurses play in bringing about those outcomes. The latter degree is a research-focused doctorate designed to advance the science behind nursing practice. As of 2019, 40,271 nurses in the U.S. held DNP degrees according to statistics generated by the American Association of Colleges of Nursing while 6,994 nurses held Ph.D.s in nursing. If you’re an advanced practice nurse who’s interested in reaching the highest echelons of your profession, the DNP vs. Ph.D. in nursing quandary may be one you find yourself thinking about frequently. Keep reading to discover 12 key differences between DNP and Ph.D. in nursing.

What is a DNP Degree?

What is a ph.d. in nursing degree, what concentrations are offered in a dnp degree, • family nurse practitioner (fnp):, • adult-gerontology nurse practitioner (agnp):, • pediatric nurse practitioner (pnp):, • neonatal nurse practitioner (nnp):, • women’s health nurse practitioner (whnp):, • psychiatric mental health nurse practitioner (pmhnp):, • certified registered nurse anesthetist (crna):, • clinical nurse midwife (cnm):, • clinical nurse specialist (cns):, • nursing education:, • executive leadership:, • nursing practice:, • health policy:, • public health:, • nursing administration:, what concentrations are offered in a ph.d. in nursing degree, • clinical nursing research:, • public health policy:, • nursing science:, • healthcare innovation:, how many years does a dnp degree take to complete, how many years does a ph.d. in nursing degree take to complete, where can you work after earning your dnp degree, where can you work after earning your ph.d. in nursing degree, how much does a dnp graduate make.

$49.86
$8,640
$103,719
(Source: Payscale.Com)

How Much Does a Ph.D. In Nursing Graduate Make?

$42.72
$7,400
$88,849
(Source: Payscale.Com)

DNP vs. Ph.D. in Nursing: What is the Difference?

1. the dnp focuses on clinical practice while the ph.d. focuses on academic research., 2. the ph.d. includes a dissertation project; the dnp includes a capstone project., 3. nursing ph.d. programs often involve a mentored teaching experience., 4. ph.d. programs prefer letters of recommendation that speak to the applicant’s academic history while dnp programs prefer letters of recommendation that speak to the applicant’s clinical experiences., 5. it takes longer to earn a nursing ph.d.., 6. dnp programs require a greater number of credits for completion of the degree., 7. dnps typically earn higher salaries than nursing ph.d.s., 8. dnps have a greater assortment of employment opportunities than nursing ph.d.s., 9. unless a nursing ph.d. is a certified nurse practitioner, his or her scope of practice is limited to bedside nursing., 10. nursing ph.d. graduates do not have to go through a periodic recertification process., 11. most universities will not offer tenure track teaching positions to dnps., 12. nursing ph.d. students are not responsible for completing a minimum number of clinical hours., conclusion - dnp vs. ph.d. in nursing: which one should you pursue.

phd nursing meaning

RN Careers

Earn Your Ph.D. in Nursing

RN-Staff-Writer

What is a Ph.D. in Nursing?

The Ph.D. in nursing produces high-caliber nurse researchers and scientists who contribute cutting-edge discoveries to inform best practices in care delivery, administration, and nursing education nationwide. It is the peak academic credential for those pursuing a career in nurse academia or scientific investigation within healthcare.

Definition of a Ph.D. in Nursing

  • A Doctor of Philosophy (Ph.D.) in Nursing is the highest academic degree awarded in the field of nursing
  • It is a research-focused doctoral program that prepares nurses to be scientists, scholars, and leaders in the discipline

The Significance of a Ph.D. in Nursing

  • Advances nursing science and evidence-based practice through rigorous research
  • Helps address pressing healthcare problems and improve patient outcomes
  • Prepares nurse experts to serve in advanced specialty, research, faculty, and leadership roles
  • Drives innovation in nursing education curriculum, policy development, and clinical systems
  • Elevates the scholarly standing and strength of the nursing profession across clinical and academic environments

Ph.D. in Nursing vs. a DNP Degree

The Ph.D. (Doctor of Philosophy) in Nursing and the DNP (Doctor of Nursing Practice ) are two distinct terminal degrees in the field of nursing, each catering to different career aspirations and areas of focus within the profession. Here’s a breakdown of the key differences between the two and the career paths typically associated with each.

Key Differences Between a Ph.D. in Nursing and a DNP

Focus and Objective:

  • Ph.D. in Nursing:  Primarily research-oriented, focusing on generating new knowledge and theories in nursing and healthcare. It prepares graduates to conduct independent research, analyze policy, and contribute to the academic foundation of nursing science.
  • DNP:  Practice-oriented, focusing on applying research to clinical practice. It prepares nurses to implement the science developed by nurse researchers in clinical settings, improve systems of care, and measure patient outcomes.

Curriculum:

  • Ph.D. in Nursing:  Emphasizes research methodologies, statistical analysis, and theoretical foundations. Students engage in dissertation work to contribute original research to the field.
  • DNP:  Centers on clinical practice, leadership, policy, and quality improvement. DNP projects often focus on translating research into practice, evaluating interventions, or improving outcomes within specific patient populations or systems.

Duration and Coursework:

  • Ph.D. in Nursing:  Typically requires more time than a DNP due to the extensive research and dissertation component. Programs can range from 4 to 6 years, depending on full-time or part-time study.
  • DNP:  Programs can be completed in 1 to 3 years, depending on the candidate’s background and the program’s structure, with some requiring clinical hours for those not already holding an advanced practice nursing degree.

Ph.d. In Nursing

Ph.D. in Nursing

Career Paths

Ph.D. in Nursing:

  • Academia:  Teaching at the university level, often in nursing programs, and conducting research.
  • Research:  Leading research projects, working in research institutions, or for healthcare policy think tanks.
  • Leadership:  High-level positions in nursing research organizations, governmental agencies, or nonprofit health organizations, focusing on policy development and implementation.
  • Advanced Clinical Practice:  Serving in roles such as Nurse Practitioners (NPs), Clinical Nurse Specialists (CNS), Nurse Anesthetists, or Nurse Midwives, with a focus on applying evidence-based practice to improve patient outcomes.
  • Healthcare Leadership and Administration:  Managing nursing departments, leading quality improvement initiatives, or holding executive positions in healthcare organizations.
  • Policy and Advocacy:  Working with governmental agencies or non-profit organizations to develop, implement, and evaluate health policies affecting patient care and nursing practice.

Ultimately, the choice between a Ph.D. in Nursing and a DNP depends on your career goals, whether you are more inclined towards academic research and teaching (Ph.D.) or applying research to improve clinical practice, lead healthcare teams, and influence health policy (DNP).

Becoming a Ph.D. in Nursing

Becoming a Ph.D. in Nursing is a rigorous academic journey that prepares nurses for careers in research, academia, and leadership, focusing on the advancement of nursing science and healthcare. The path from a Bachelor of Science in Nursing (BSN) to a Ph.D. in Nursing involves several steps, each with its own set of prerequisites and decisions regarding enrollment status.

BSN to Ph.D. in Nursing: The Academic Journey

1. Earning a Bachelor’s Degree:

  • The journey begins with obtaining a BSN. This foundational step provides the clinical skills and theoretical knowledge necessary for advanced study in nursing.

2. Gaining Clinical Experience (Optional but Recommended):

  • While not always a requirement for Ph.D. programs, gaining practical nursing experience can enhance understanding of the healthcare system and inform future research interests.

3. Research Experience:

  • Engaging in research activities during or after the BSN program can be beneficial. This experience can include working on research projects, attending workshops, and getting involved in academic publications.

4. Selecting a Program:

  • Choose a Ph.D. program that aligns with your research interests and career goals. Consider faculty expertise, program curriculum, and available resources.

5. Master’s Degree (Optional):

  • Some students may pursue a Master’s degree in nursing or a related field before entering a Ph.D. program, although many Ph.D. programs accept students directly from a BSN background and may offer a combined MSN-Ph.D. track.

Prerequisites for a Ph.D. in Nursing

1. Educational Background:

  • A BSN or, in some cases, a Master’s degree in nursing or related field. Some programs allow direct entry from a BSN, while others require an MSN .

2. Academic Achievement:

  • High undergraduate GPA, usually a minimum of 3.0 on a 4.0 scale.
  • Completion of graduate-level statistics course is often required.
  • Experience with research methodologies, data analysis, or involvement in research projects is highly recommended.

4. Letters of Recommendation:

  • Letters from academic and professional references attesting to the applicant’s potential for doctoral study and research.

5. Statement of Purpose:

  • A detailed essay outlining research interests, career goals, and reasons for pursuing a Ph.D. in Nursing.

6. GRE Scores:

  • Some programs require Graduate Record Examination (GRE) scores, although this requirement is increasingly being waived.

Full-time vs. Part-time Enrollment Options

Full-time Enrollment:

  • Typically requires a commitment of 3-5 years.
  • More intensive, allowing for quicker progression through the program.
  • May offer more opportunities for funding, such as scholarships, fellowships, and assistantships.

Part-time Enrollment:

  • Extends the duration of the program, often taking 5-7 years or more.
  • Offers flexibility for students who need to work or fulfill other obligations.
  • May limit access to certain funding opportunities.

Both full-time and part-time enrollment have their advantages and challenges. The choice depends on personal circumstances, financial considerations, and career objectives. Some students prefer the immersive experience and faster completion of full-time study, while others require the flexibility of part-time enrollment to balance academic, professional, and personal responsibilities.

PhD in Nursing Programs

When considering the best Ph.D. in Nursing programs globally, it’s essential to look at several factors, including the quality of research, faculty expertise, available resources, and the program’s reputation. While specific rankings can vary by publication and criteria, institutions known for their nursing and healthcare programs often stand out due to their contributions to research, policy, and practice in nursing.

Best Ph.D. in Nursing Programs Globally

Some of the top institutions known for their nursing Ph.D. programs include:

  • Johns Hopkins University School of Nursing  (USA): Known for its research contributions and leadership in healthcare, offering a curriculum that emphasizes innovative research methodologies, evidence-based practice, and healthcare policy.
  • University of Pennsylvania School of Nursing  (USA): Offers a research-intensive Ph.D. program focusing on improving healthcare across the lifespan, with faculty members leading in various fields of nursing science.
  • University of Toronto Lawrence S. Bloomberg Faculty of Nursing  (Canada): Recognized for its strong research programs, diverse specializations , and contributions to healthcare policy and practice.

These programs are renowned for their rigorous curriculum, research opportunities, and the ability to produce graduates who go on to become leaders in nursing research, education, and policy.

Online Ph.D. Programs in Nursing and Their Credibility

Online Ph.D. programs in Nursing have become increasingly popular, offering flexibility for working professionals. The credibility of these programs depends on their accreditation, faculty qualifications, and the institution’s reputation. Accredited online Ph.D. programs in Nursing provide education and research opportunities comparable to their on-campus counterparts. Key factors to consider include:

  • Accreditation:  Ensure the program is accredited by a recognized accrediting body, such as the Commission on Collegiate Nursing Education (CCNE) or the Accreditation Commission for Education in Nursing (ACEN) in the United States.
  • Faculty Expertise:  Look for programs with experienced faculty involved in active research and with strong professional backgrounds in nursing.
  • Resources and Support:  High-quality online programs offer access to digital libraries, research databases, and strong support systems for online students.

Specializations within Nursing Ph.D. Programs

Ph.D. in Nursing programs often offer specializations that allow students to focus on areas of interest within the field. These specializations can include:

  • Nursing Informatics :  Focuses on the integration of nursing science with information management and analytical sciences to improve health outcomes.
  • Psychiatric Nursing :  Concentrates on the care of patients with mental health issues, including research into psychiatric treatments and interventions.
  • Anesthesia Nursing:  Although more commonly associated with Nurse Anesthesia programs (DNAP), some Ph.D. programs might focus on research in anesthesia practices and outcomes.
  • Mental Health Nursing :  Involves research aimed at improving the mental health care system and outcomes for patients with mental health conditions.

Choosing a specialization depends on your career goals, research interests, and the healthcare needs you are most passionate about addressing. Each specialization offers a path to contribute significantly to the nursing profession and healthcare at large, whether through advancing clinical practice, shaping health policy, or improving educational methodologies.

Application Process

Applying for a Ph.D. in Nursing involves a detailed and often competitive process. Here’s a general guide on how to apply and fulfill the entrance requirements for these programs:

1. Research and Select Programs

  • Identify Programs:  Begin by researching programs that align with your career goals and research interests. Consider factors such as faculty expertise, research facilities, funding opportunities, and program format (on-campus vs. online).
  • Check Accreditation:  Ensure the program is accredited by a recognized accrediting body for nursing education, such as the Commission on Collegiate Nursing Education (CCNE) or the Accreditation Commission for Education in Nursing (ACEN) in the U.S.

2. Fulfill Prerequisites

  • Educational Background:  Most programs require applicants to have a Bachelor of Science in Nursing (BSN) or a Master of Science in Nursing (MSN). Some programs offer BSN-to-Ph.D. tracks.
  • Academic Records:  You’ll need to provide transcripts from all post-secondary institutions attended. A high GPA, usually above 3.0 on a 4.0 scale, is often required.
  • Research Experience:  Engage in research activities, which can include working on research projects, publishing papers, or attending research workshops, to strengthen your application.
  • Professional Experience:  While not always mandatory, relevant nursing or healthcare experience can enhance your application.

3. Prepare Application Materials

  • Letters of Recommendation:  Obtain letters from academic and professional references who can attest to your potential for doctoral study and research.
  • Statement of Purpose:  Write a compelling essay outlining your research interests, why you want to pursue a Ph.D. in Nursing, and why you’re applying to specific programs.
  • Curriculum Vitae (CV):  Include your educational background, research experience, publications, professional experience, and any honors or awards.
  • GRE Scores:  Some programs may require GRE scores, though this requirement is increasingly being waived. Check the specific requirements of each program.

4. Complete Application Forms

  • Fill Out Applications:  Complete the application forms for each program you’re applying to. Pay attention to each program’s specific requirements and deadlines.
  • Application Fee:  Be prepared to pay an application fee for each program.

5. Submit Applications

  • Review Your Application:  Before submitting, double-check your application for completeness and accuracy.
  • Submit Before Deadlines:  Ensure all parts of your application, including letters of recommendation and test scores (if required), are submitted before the program’s deadline.

6. Prepare for Interviews

  • Some programs may invite applicants for interviews as part of the selection process. These interviews can be in-person or virtual. Prepare to discuss your research interests, professional goals, and how the program fits your career aspirations.

7. Follow Up

  • After submitting your application, monitor your email and application portal for updates or requests for additional information.

Fulfilling Entrance Requirements

To fulfill the entrance requirements:

  • Advance Planning:  Begin planning your application well in advance of deadlines. This includes researching programs, understanding prerequisites, and preparing application materials.
  • Strengthen Your Profile:  Engage in activities that will strengthen your application, such as gaining research experience, publishing work, and participating in professional development opportunities.
  • Seek Guidance:  Consider seeking advice from mentors, faculty members, or professionals in the field who can provide insights into the application process and how to strengthen your application.

Applying for a Ph.D. in Nursing is a significant step towards advancing your career in nursing science and leadership. By carefully preparing your application and aligning your goals with the strengths of the programs you choose, you can increase your chances of admission to a program that will help you achieve your professional aspirations.

Online Nursing Degree

Online Ph.D. in Nursing programs provide a flexible pathway for nurses to pursue the highest level of nursing education without the need to attend traditional on-campus classes. These programs are designed to accommodate the schedules of working professionals, offering a blend of synchronous (live) and asynchronous (pre-recorded lectures and materials) coursework, virtual interactions with faculty and peers, and digital access to library resources and research materials.

Overview of Online Ph.D. in Nursing Programs

Online Ph.D. programs in Nursing focus on preparing nurses for roles in research, academia, leadership, and policy. The curriculum typically includes advanced courses in research methods, statistics, nursing theory, healthcare policy, and ethics, alongside specialized courses that align with the student’s research interests and career goals. Students also undertake a dissertation project, which involves original research contributing to nursing science.

Key components of online Ph.D. in Nursing programs include:

  • Virtual Classroom Sessions:  For lectures, discussions, and presentations.
  • Digital Access to Research Resources:  Including online journals, databases, and libraries.
  • Online Collaboration Tools:  For group projects and peer interaction.
  • Virtual Advising and Mentoring:  For research guidance and academic support.
  • Dissertation Defense:  Often conducted virtually or may require a campus visit.

Advantages and Considerations of Online Learning for a Ph.D.

Advantages:

  • Flexibility:  Allows students to balance their studies with professional and personal commitments.
  • Accessibility:  Enables students from anywhere in the world to access top-quality programs without the need for relocation.
  • Technology Skills:  Enhances digital literacy and familiarity with online research tools and collaboration platforms.

Considerations:

  • Self-Discipline and Motivation:  Success in an online program requires a high degree of self-discipline and the ability to work independently.
  • Limited In-Person Interaction:  Although technology facilitates interaction, the lack of face-to-face contact may be a drawback for some students.
  • Technology Requirements:  Students must have access to reliable internet and the necessary hardware and software to participate in online coursework.

Accredited Online Ph.D. Programs in Nursing

When considering an online Ph.D. in Nursing program, accreditation is crucial to ensure the program meets high standards of quality and rigor. Accredited programs are recognized for their excellence in education, faculty qualifications, and student support services, making them more respected by employers and other institutions.

Some accredited online Ph.D. in Nursing programs include:

  • Johns Hopkins University School of Nursing:  Offers a Doctor of Philosophy in Nursing (Ph.D.) that prepares nurse scientists to use evidence-based practice in teaching and research.
  • University of Illinois at Chicago College of Nursing:  Provides a Ph.D. in Nursing program focused on research that improves nursing education, practice, and policy.
  • Duke University School of Nursing:  Offers a Ph.D. program designed to develop nursing scholars capable of advancing health care through research.

It’s essential to verify the accreditation status of any program you consider by checking with recognized accrediting agencies such as the Commission on Collegiate Nursing Education (CCNE) or the Accreditation Commission for Education in Nursing (ACEN) in the United States.

Choosing an online Ph.D. in Nursing program requires careful consideration of your career goals, learning style, and the specific features and accreditation of the programs you’re considering. With the right preparation and commitment, an online Ph.D. in Nursing can provide a flexible and rigorous path to advance your nursing career to the highest level.

Coursework and Research

A Ph.D. in Nursing is designed to prepare nurses for advanced roles in research, academia, leadership, and policy development. The program typically combines coursework, research, and a dissertation to provide a comprehensive understanding of nursing science and research methodologies. Below is an overview of the typical curriculum, research expectations, and dissertation requirements for a Ph.D. in Nursing.

Typical Curriculum and Coursework

The curriculum for a Ph.D. in Nursing focuses on developing advanced research skills, theoretical knowledge, and a deep understanding of issues affecting healthcare and nursing practice. While specific courses may vary by program, the following are commonly included:

  • Advanced Research Methods:  Courses covering qualitative and quantitative research methodologies, data analysis techniques, and research design.
  • Statistics:  Instruction in advanced statistical methods for analyzing research data.
  • Nursing Theory:  Exploration of theoretical frameworks in nursing and their application to research and practice.
  • Healthcare Policy:  Examination of policy-making processes, health care systems, and the impact of policy on patient care and nursing practice.
  • Ethics in Research and Healthcare:  Discussion of ethical considerations in nursing research and clinical practice.
  • Specialized Electives:  Courses tailored to students’ research interests, such as informatics, gerontology, public health, or education.

The Research Component: Expectations and Opportunities

Research is a cornerstone of the Ph.D. in Nursing, with students expected to contribute original knowledge to the field. Expectations and opportunities include:

  • Research Mentorship:  Students work closely with faculty mentors who provide guidance on research projects, methodology, and dissertation development.
  • Research Projects:  Participation in ongoing research projects, often as part of a team, to gain practical experience and contribute to scholarly work.
  • Conferences and Publications:  Opportunities to present research findings at conferences and publish in academic journals, which are crucial for professional development and visibility in the field.

Dissertation Requirements and the Process of Completion

The dissertation is a significant component of the Ph.D. program, demonstrating the student’s ability to conduct independent research and contribute to nursing science. The process typically involves:

  • Proposal Development:  Students develop a research proposal, including a literature review, research question, methodology, and proposed analysis. This proposal is presented to a faculty committee for approval.
  • Conducting the Research:  After receiving approval, students conduct their research, collect data, and analyze results under the guidance of their advisor and committee.
  • Writing the Dissertation:  The findings are written up in a dissertation, which includes an introduction, literature review, methodology, results, discussion, and conclusion.
  • Defense:  The completed dissertation is presented and defended before a committee of faculty members. The defense includes a presentation of the research findings followed by questions from the committee.

Completion of the dissertation process requires meticulous planning, research, and writing skills. It can take several years, depending on the complexity of the research, the student’s pace of work, and other commitments. Successful defense of the dissertation is a significant achievement, marking the culmination of the Ph.D. journey and the student’s readiness to contribute to the nursing profession as a researcher and academic leader.

Duration and Commitment

A PhD in nursing demands nearly full-time effort, though some maintain nursing instructor roles or carefully-balanced side-jobs during non-intensive coursework terms. Planning ahead financially allows immersive focus for the 1-2 years spent progressing through the doctoral dissertation research phase, widely considered the most demanding but rewarding undertaking of PhD training.

How Long Does It Take to Get a PhD in Nursing

  • Full-time enrollment: Around 4-6 years
  • Part-time enrollment: 5-8 years on average
  • 2 years coursework
  • Candidacy exam
  • 1-2 years for dissertation research
  • Some accelerated 3-year PhD options through full-time accelerated study

Balancing Work and Study: Is This Feasible?

  • Extremely challenging to balance full-time work and PhD studies
  • Research and dissertation writing are rigorous full-time endeavors
  • Most PhD nursing programs strongly recommend not working more than part-time during studies
  • Can maintain part-time clinical work such as teaching to promote career development
  • Finances need to be considered to enable focusing completely on PhD research requirements

Financial Aspects

Careful financial planning and taking advantage of available nursing PhD funding sources helps balance the high costs of advanced nursing education – costs that pay dividends over the course of an accomplished research and/or academic career.

PhD in Nursing Salary Expectations and Influencing Factors

  • Average salary: $100,000+
  • Years of experience
  • Faculty rank
  • Type of university
  • Geographic location
  • Significant earning potential over MSN or DNP alone

Scholarships and Funding Opportunities for PhD Students

  • Research or teaching assistantships
  • Private foundations, professional associations
  • NRSA Grants from National Institutes of Health
  • U.S. Health Resources and Services Administration
  • Military nursing scholarships

Cost of PhD Programs and Return on Investment

  • Tuition ranges from $20,000 – $50,000+ total
  • Additional costs like books, travel, living expenses
  • High ROI – earning potential far exceeds investment
  • Many utilize student loans, grants, scholarships
  • Graduates see salary bump of 25-50%

Career Opportunities and Outcomes

A PhD in nursing prepares graduates to significantly expand nursing’s impact – whether leading transformative research, training future generations of nurses, implementing improvements across healthcare organizations, or informing policy through expansive perspectives on urgent challenges for practice and patient populations.

Jobs Available for Those with a PhD in Nursing

  • Nurse scientist roles
  • Professorships at universities
  • Program research directors
  • Executive nursing administrators
  • Consultant positions
  • Grant writing specialists
  • Editors of academic journals

How a PhD in Nursing Can Enhance Your Career

  • Opens access to top academic and healthcare leadership roles
  • Enables spearheading innovative nursing research at prestigious institutions
  • Qualifies graduates to teach in bachelor’s and graduate nursing programs
  • High demand for PhDs to fill new faculty job openings
  • Greater career satisfaction and sense of purpose

Leadership Roles in Nursing and Contributions

  • Develop evidence-based policies, programs, best practices
  • Consulting governments, healthcare systems, private sector organizations
  • Ensuring nurse education curriculum reflects latest advances
  • Evaluating effectiveness of nursing care approaches
  • Publishing pioneering studies and theories in nursing journals
  • Elevating quality standards across academia and practice

What Are the Roles and Duties of a Ph.D. Nurse?

Nurses who earn a Ph.D. are prepared for a variety of roles within academia, research, leadership, and healthcare policy. Their duties can vary significantly depending on their specific job title and the setting in which they work. Here’s a look at the roles and typical duties of Ph.D. nurses in three key areas: research, education, and leadership/administration.

Nurse Researcher

Nurse researchers with a Ph.D. are vital in advancing nursing science, improving patient care, and influencing health policy. Their typical duties include:

  • Conducting Original Research:  Designing and implementing studies to generate new knowledge relevant to nursing practice, patient outcomes, and healthcare systems.
  • Securing Funding:  Writing grant proposals to obtain funding from governmental agencies, non-profit organizations, and private entities to support research projects.
  • Data Analysis:  Employing statistical techniques to analyze research data and interpret findings.
  • Publishing Findings:  Writing articles for publication in scholarly journals, presenting research at conferences, and disseminating results to the healthcare community and the public.
  • Collaborating:  Working with other healthcare professionals, researchers, and stakeholders to conduct interdisciplinary research and implement findings in clinical practice.

Ph.D. Nurse Educator

Ph.D. nurse educators play a crucial role in shaping the future of nursing through teaching, curriculum development, and mentorship. Their duties include:

  • Teaching:  Delivering lectures, seminars, and workshops to undergraduate, graduate, and doctoral nursing students.
  • Curriculum Development:  Designing, evaluating, and updating nursing education programs and courses to reflect current research, best practices, and regulatory standards.
  • Mentoring:  Guiding students through their academic journey, providing research opportunities, and advising on career paths.
  • Scholarly Activities:  Engaging in research, publishing articles, and presenting at academic conferences to contribute to the nursing body of knowledge.
  • Academic Leadership:  Participating in faculty governance, accreditation processes, and program evaluation to ensure the quality and integrity of nursing education.

Department Chair or Administration

Nurses with a Ph.D. who move into department chair or administrative roles take on additional responsibilities related to the management and strategic direction of nursing programs or healthcare organizations. Their duties may include:

  • Strategic Planning:  Developing long-term goals and strategies for the nursing department or educational program.
  • Budget Management:  Overseeing the budget, allocating resources efficiently, and securing funding for departmental needs.
  • Faculty Management:  Hiring, evaluating, and mentoring faculty members, and fostering a supportive and productive work environment.
  • Program Development:  Leading the development and accreditation of new programs or specializations to meet emerging healthcare needs.
  • Policy Implementation:  Establishing policies and procedures to enhance the educational quality, research output, and clinical practices within the organization.
  • Stakeholder Engagement:  Representing the nursing department in discussions with university administration, healthcare partners, and external stakeholders to advocate for nursing education and practice.

Ph.D. nurses, regardless of their specific role, contribute significantly to the nursing profession through leadership, innovation, and a commitment to advancing healthcare outcomes. Their work not only impacts the immediate settings in which they operate but also influences the broader scope of healthcare policy, nursing practice, and patient care globally.

FAQ on DNP and Ph.D. in Nursing

Q1: What are the main differences between a DNP and a Ph.D. in Nursing? A1: The main difference lies in the focus and outcome of each program. A DNP (Doctor of Nursing Practice) is practice-oriented, focusing on applying research to clinical practice, improving healthcare systems, and enhancing patient outcomes. A Ph.D. (Doctor of Philosophy) in Nursing is research-oriented, aiming to generate new knowledge in nursing and health through original research. Ph.D. programs often emphasize theoretical foundations of nursing, research methodologies, and teaching nursing, while DNP programs concentrate on clinical practice, leadership, and health care administration.

Q2: What is a Doctor of Nursing Science (DNSc) and how does it compare to a DNP or Ph.D.? A2: A Doctor of Nursing Science (DNSc) is another doctoral degree focused on the advanced study of nursing, often with a strong emphasis on research and the theoretical foundations of nursing, similar to a Ph.D. The DNSc might be more research-oriented than a DNP but more applied than a Ph.D., depending on the program. However, DNP programs have become more prevalent in preparing advanced practice registered nurses (APRNs) and leaders in clinical settings.

Q3: How long does it typically take to complete a DNP or Ph.D. in Nursing? A3: The duration can vary based on the program and whether the student is enrolled full-time or part-time. DNP programs can typically be completed in three to five years of full-time study, while Ph.D. programs may take longer, often requiring five years of full-time study due to the research and dissertation component.

Q4: Are online nursing programs credible for pursuing a DNP or Ph.D. in Nursing? A4: Yes, many credible and accredited online nursing programs are available for both DNP and Ph.D. degrees. These programs offer flexibility for working professionals but require the same level of rigor and coursework as traditional on-campus programs. Accreditation by reputable organizations, such as the Commission on Collegiate Nursing Education (CCNE), is key to ensuring the program’s credibility.

Q5: What kind of nursing courses can I expect in a DNP degree program? A5: DNP degree programs generally include courses in advanced clinical practice, leadership and management, health policy, evidence-based practice, and quality improvement. The exact curriculum can vary from school to school, but all aim to prepare nurses for the highest level of clinical nursing practice.

Q6: What are the prerequisites for enrolling in a DNP or Ph.D. in Nursing program? A6: Prerequisites can vary, but typically, applicants to DNP programs must hold a master’s degree in nursing or a related field, have a valid RN license, and some programs may require a certain number of clinical practice hours. For Ph.D. programs, requirements often include a bachelor’s or master’s degree in nursing, academic transcripts demonstrating a strong academic record, research experience, and letters of recommendation. Some Ph.D. programs may also require GRE scores.

Q7: Can students choose specializations within DNP or Ph.D. in Nursing programs? A7: Yes, students can choose from various specializations depending on the program’s offerings. These can include areas like family nurse practitioner, nurse anesthesia, nursing informatics, psychiatric-mental health nursing, and health care administration for DNP programs. Ph.D. programs may offer specializations in areas such as nursing education, health policy, or specific research methodologies.

Q8: What are the career prospects for nurses with a DNP or Ph.D.? A8: Nurses with a DNP are prepared for clinical leadership roles, such as nurse practitioners, clinical nurse specialists, nurse executives, and roles in health care administration. Nurses with a Ph.D. often pursue careers in academic teaching, research, and scholarly writing. According to the Bureau of Labor Statistics, employment for advanced practice registered nurses is projected to grow, indicating strong prospects for DNP graduates. Ph.D. graduates contribute to the nursing profession’s body of knowledge and help educate the next generation of nurses.

Q9: Is earning a DNP or Ph.D. necessary for advancement in the nursing profession? A9: While not necessary for all advancement opportunities, earning a DNP or Ph.D. can significantly enhance a nurse’s career prospects, allowing for roles in advanced clinical practice, academia, research, and leadership that are not accessible with lower levels of education. These degrees prepare nurses for the highest degree of professional practice and contribution to healthcare improvements and nursing knowledge.

  • Sigma Theta Tau International: https://www.sigmanursing.org/
  • National Student Nurses Association: https://www.nsna.org/
  • National League for Nursing: http://www.nln.org/
  • The American Board of Nursing Specialties: http://www.nursingcertification.org/
  • Emergency Nurses Association: https://www.ena.org/
  • American Association of Critical-Care Nurses: https://www.aacn.org/
  • Best MSN to DNP Programs in Each State
  • Best Nursing Schools in Delaware - 2024
  • 100 Trailblazing Nurses Who Paved the Way for Modern…
  • How to Become a Travel Nurse: Salary and Requirements
  • 10 Best RN to BSN Programs in Virginia 2024
  • Nursing Schools in Wyoming
  • About the Author
  • Latest Posts

RN-Staff-Writer

After graduating with a degree in English Literature from UCLA, Jeff published four editions of the World Wide Web Yellow Pages by Barnes and Noble, and several editions of the Best of the Web. He’s worked as an executive at both startup and mature companies including CareerPath.com and Microsoft. With nearly three decades of publishing and business development expertise, he now puts that experience to use operating RNCareers.org to help future nursing students get reliable information on registered nursing careers.

  • Best Nursing Schools in the US
  • Best Online Nursing Schools in the United States
  • MBA in Nursing Jobs: 10 Lucrative Careers

Similar Posts

Become An Adn

Associates Degree in Nursing

Lauren Jacobson, RN

Your 2024 Comprehensive Guide to Nursing Careers

Taylor Haskings

Paramedic to RN Bridge Programs: Online RN Options for 2024

Benefits Of Becoming A Nurse Practitioner

Earn Your CNA Certification

Paige Shreffler RN Nurse

Master’s of Science in Nursing – MSN Degree

Doctor Of Nursing Practice (Dnp)

Become a Doctor of Nursing Practice (DNP)

RNCareers

Let’s Get in Touch

At RNCareers.org, you are teaming up with our team of nurse educators, nurse practitioners, DNPs, RNs LPNs and students to learn about nursing careers, programs and salaries for nurses.

Headquarters, California

Monday – Friday, 9:30 am – 5:30 pm

US Team :  Agoura Hills, CA; Cumming, GA; Beverly Hills, CA; Marquette, MI; Lilburn, GA; Terre Haute, IN; Pittsburgh, PA; Loma Linda, CA; Carlsbad, CA

Worldwide Team : Salford, United Kingdom; Cairo, Egypt; Novovolynskiy, Ukraine; Ahmedabad, India; Alexandria, Egypt; Kushtia, Bangladesh; Kyiv, Ukraine; Toronto, Canada

Certificate Bachelor ABSN Master Doctorate

Highest Paying

Nurse Anesthetist Nurse Practitioner Nurse Midwife Nurse Researcher Clinical Nurse Specialist

Salary Reports

CNA LPN Instructor RN Midwife NP CRNA

SALARIES BY STATE

California Texas Florida New York Pennsylvania Illinois Ohio Georgia North Carolina Mic h igan

© 2009 – 2024 RNCareers.org –   About  |  Contact  |  Experts  |  Data Sources  |   Sitemap  |  Privacy  |  Terms of Service  |  Advertising Disclosure  | Cookie Policy |  Do Not Sell My Information | Ranking Methodology

University of St. Augustine for Health Sciences

University Administration

  • Careers @ USA
  • For Current Students
  • Request Information
  • Master of Occupational Therapy (MOT)
  • Doctor of Occupational Therapy (OTD)
  • Post-Professional Doctor of Occupational Therapy (PPOTD)
  • Master of Health Administration (MHA)
  • Doctor of 
Education (EdD)
  • Graduate Certificates
  • Clinical Orthopedic Residency (OCS)
  • Orthopaedic Manual Physical Therapy Fellowship (OMPT)
  • Continuing Professional Education (CPE)
  • Doctor of Physical Therapy (DPT)
  • Non-Degree Physical Therapy Online Courses
  • Master of Science in Speech-Language Pathology (MS-SLP)
  • Post-Graduate Nursing Certificates
  • Master of Science in Nursing (MSN)
  • Doctor of Nursing Practice (DNP)
  • Browse All Degree-Programs
  • Admissions & Aid Home
  • Scholarships & Grants
  • How to Apply
  • Cost of Attendance
  • Financial Aid
  • Application Deadlines
  • Academic Calendar
  • Financial Aid FAQ
  • Admissions FAQs
  • Catalog/Handbook
  • Our History
  • Accreditation
  • B Corp Certified
  • Change of Ownership
  • Student Achievement Data
  • Institutional Learning Outcomes (ILOs)
  • Diversity & Inclusion
  • San Marcos, California
  • St. Augustine, Florida
  • Miami, Florida
  • Austin, Texas
  • Dallas, Texas
  • Our Faculty
  • Board of Directors

University of St. Augustine for Health Sciences

Nursing MSN & DNP

| 18 December 2023

PhD in Nursing vs. DNP: Which Is Right for You?

A USAHS student in the Immersion Lab.

Nurses don’t just care for patients—they can also advance to leadership positions that shape everything from patient care to healthcare policy. We break down everything you need to know about a PhD in Nursing vs. a DNP so you can pick the best degree for your career goals.

Table of Contents

What Is a PhD in Nursing?

What is a dnp, dnp vs. phd in nursing: main differences.

A PhD in nursing is one of two terminal degrees in the field, representing the highest level of education possible for nurses. 1 A PhD in nursing is a research and science-focused degree, so it centers on preparing nurses for careers involving medical research to advance the entire nursing profession or for teaching nursing at the college level rather than clinical practice.

Definition of a PhD in nursing.

How To Earn a PhD in Nursing

To apply to a PhD in Nursing program:

  • Earn your Bachelor of Science in Nursing (BSN) from an accredited institution. 1 This degree prepares you with foundational knowledge on topics such as anatomy, pharmacology, pathophysiology and more. 2 This generally takes four years to earn. 3
  • Pass the National Council of Licensure Examination (NCLEX-RN). All nurses must pass the NCLEX-RN to practice in the United States. 4 The exam assesses your critical thinking skills and knowledge of nursing practices.
  • Earn your Master of Science in Nursing (MSN). Some programs may require an MSN for admission into their program. 1 An MSN is a graduate degree focused on specific nursing topics to prepare you for advanced practice or nursing leadership. You can choose from various specializations, including Family Nurse Practitioner, Psychiatric Mental Health Nurse Practitioner, Nurse Executive or Nurse Educator.

Once you’ve completed these steps, begin applying to PhD in nursing programs.

Curriculum Expectations

The specific courses you’ll take in a PhD in nursing program will vary by program, but you can expect your coursework to focus on the following 5 :

  • History and philosophy of nursing
  • Development and testing of nursing and other healthcare techniques
  • Data management
  • Research methodology
  • Social, economic, political and ethical issues in nursing

You’ll also need to complete a dissertation focused on original research. 1

Career Outlook

Earning a PhD in Nursing qualifies you for two specific career pathways: research and nursing education. 1

A nurse researcher may work in a hospital, medical lab or other research facility, where they design and perform scientific research to propel the nursing profession forward. 1 Their duties include:

  • Craft research questions and write research proposals
  • Conduct research to answer those questions
  • Collaborate with other scientists in nursing and other related medical fields
  • Collect and analyze data
  • Publish their findings
  • Apply for grants and funding for research proposals
  • Create and follow quality assurance standards
  • Train and supervise laboratory staff, including nurses and scientists.

A nurse educator works at universities and colleges to train nursing students. 1 Their day-to-day responsibilities include the following:

  • Oversee course curriculum
  • Teach courses for undergraduate and graduate nursing students
  • Supervise students’ laboratory and clinical work
  • Grade student work and rate their performance
  • Mentor and advise nursing students

Benefits of Earning a PhD in Nursing

Earning your PhD in nursing has plenty of benefits that make it appealing to nurses wishing to advance in the profession:

  • A more traditional work schedule. 6 Nurses often work 12-hour shifts, including overnights, weekends and holidays. Academia, however, sticks to more traditional work hours and adheres to national holidays.
  • The opportunity to influence healthcare policy and practice. 7 Working in academia, you can design and conduct research whose findings could potentially change healthcare policies and nursing practice.
  • Administrative and leadership opportunities. 7 A PhD in Nursing provides leadership opportunities in research and academia that a BSN or MSN wouldn’t qualify you for. You can also lead development nurses as they enter the profession.

A Doctor of Nursing Practice (DNP) prepares you for the highest level of nursing expertise in a clinical setting in addition to management and leadership roles. 8 You do not have to practice with a DNP, but it does give you the option.

Definition of a DNP.

How To Earn a DNP

The journey to earn your DNP may vary based on where you start. At USAHS, the path to your DNP follows these steps:

  • Earn a Bachelor of Science in Nursing (BSN). You’ll complete a four-year program consisting of traditional liberal arts courses, general electives and science courses. While you can start your nursing career with an Associate Degree in Nursing (ADN), you’ll need a BSN to qualify for a DNP program. If you have a BSN, you may be able to enter a BSN-entry DNP nursing program , which will include selecting a role specialty.
  • Pass the NCLEX-RN exam. Depending on your state, you must pass the NCLEX-RN exam within three years of graduating.
  • Submit a transcript that shows an accredited school conferred a nursing degree.
  • Share your NCLEX exam results.
  • Complete whatever additional paperwork your state requires.
  • Pay an application fee, which averages around $100. 9
  • Pass a criminal background check.
  • Gain work experience. Our DNP program requires applicants to have a minimum of one year of nursing experience as an RN by the time they begin their first practicum course. This on-the-job experience builds your skills and allows you to show career progression in responsibilities, titles or skill sets.
  • Earn a Master of Science in Nursing (MSN). An MSN is a graduate nursing program that prepares you for leadership roles and provides specialized training. If you are applying to USAHS, this step is optional since you can apply with a BSN.
  • Your entry point (BSN or MSN)
  • Area of specialization
  • Program requirements
  • Acceleration or extension opportunities
  • Whether you are enrolled full- or part-time

The majority of the curriculum in a DNP program is practice-focused and will include clinical practicums and a final project similar to a dissertation. 8 , 10

The curriculum for a DNP varies slightly based on the program, your entry path and your specialization.

At USAHS, you can choose between two paths to enter the DNP program — BSN-entry or MSN-entry. If you enter the program with a BSN, you’ll earn anywhere from 64-71 credit hours and participate in 1,000-1,035 practicum hours depending on whether you enroll in the Family Nurse Practitioner or Nurse Executive pathway. Students entering the MSN pathway will earn 42-52 credits and anywhere from 540-1,000 practicum hours Please note the DNP-FNP program typically takes 3 years and 8 months to complete, based off the most recent 2-year graduate data (2020-2021 and 2021-2022); however, individual experiences will vary based on factors including, but not limited to individual progress, traditional vs. accelerated pathway, credits transferred, and other factors. .

By the end of USAHS’ DNP program , you will be able to do the following:

  • Engage in lifelong learning through intellectual inquiry and scholarship.
  • Communicate effectively with various members of a healthcare team, from patients to national leaders.
  • Lead and serve on interprofessional teams to improve population health and outcomes.
  • Utilize transformational leadership, systems thinking and clinical analysis to improve healthcare outcomes for all.
  • Practice critical thinking and evidence-based decision-making in delivering or directing nursing practice and serving as a leader in healthcare systems.
  • Provide leadership in the use of nursing informatics to improve outcomes at the patient and institutional levels.
  • Deliver or direct nursing practice based on sound theoretical frameworks, best scientific evidence, clinical expertise and in collaboration with the patient and family.
  • Exhibit professional and ethical standards while advocating for safe, equitable and cost-effective healthcare and policies for all.

Specialties

DNP students can specialize in any of the following areas 8 , 11 :

  • Adult-Gerontology Primary Care Nurse Practioner (AGPCNP)
  • Certified Registered Nurse Anesthetist (CRNA)
  • Certified Nurse-Midwife (CNM)
  • Family Nurse Practitioner (FNP)
  • Neonatal Nurse Practitioner (NNP)
  • Psychiatric Mental Health Nurse Practitioner (PMHNP)
  • Women’s Health Nurse Practitioner (WHNP)
  • Clinical Nurse Specialist (CNS)
  • Healthcare Policy
  • Nurse Executive (NE)/Nurse Administrator
  • Nurse Educator
  • Nursing Informatics

You have several career options with a DNP, including nurse practitioner and other clinical and leadership positions. You can qualify for many of these positions with an MSN degree, but earning a DNP could lead to greater financial compensation. 12

If you wish to work in direct patient care, you can use your DNP as a nurse practitioner (NP). NPs usually specialize in one of the following areas:

  • Family: A Family Nurse Practitioner (FNP) is an autonomous clinician who assesses, diagnoses and treats patients and families like a family doctor. They work with children and adults in clinics, private practices and hospitals.
  • Adult Gerontology: An Adult-Gerontology Primary Care Nurse Practitioner (AGPCNP) works with individuals, from young adults to seniors, to assess, diagnose, treat and manage illnesses and injuries. 13 They can work in clinics, private practices, hospitals and long-term healthcare facilities like nursing homes.
  • Neonatal: A Neonatal Nurse Practitioner (NNP) provides care to premature babies while supporting and educating their parents or guardians. 14 They often work in emergency rooms, delivery rooms and outpatient facilities that work with infants.
  • Pediatric: A Pediatric Nurse Practitioner (PNP) is similar to a neonatal NP, except they work with children from infancy to their late teens. 15 Since children cannot make medical decisions for themselves, pediatric NPs work closely with their parents in clinics, private practices and hospitals.
  • Psychiatric-Mental Health: A Psychiatric-Mental Health Nurse Practitioner (PMHNP) works with patients experiencing mental illness or addiction. 16 They diagnose, treat and evaluate patients in hospitals, outpatient facilities, clinics and private practices.
  • Women’s Health: A Women’s Health Nurse Practitioner (WHNP) focuses on helping patients with reproductive, obstetric and gynecological care, similar to a gynecologist. 17

Someone with a DNP who wishes to work directly with patients can also pursue the following careers:

  • Certified Nurse-Midwife (CNM): A CNM focuses on providing healthcare to women from pregnancy through postpartum. 18 They work in hospitals, birthing centers and public health clinics and hold a certification from the American Midwifery Certification Board (AMCB).
  • Certified Registered Nurse Anesthetist (CRNA): CRNAs are advanced-practice nurses who administer anesthesia during procedures and surgeries. 19 They treat patients of all ages and can work alone or with anesthesiologists at hospitals, dentist offices and other healthcare providers.
  • Clinical Nurse Specialist (CNS): A CNS supervises and collaborates with other nurses and healthcare team members, conducts research and other evidence-based projects, consults with healthcare organizations to improve system and patient outcomes and teaches or advises future nurses. 20

Nurses with DNPs do not have to work directly with patients—they can also hold many leadership and administrative roles:

  • Nurse manager: A nurse manager works administratively, overseeing staffing, personnel, human resources, financial resources, operational efficiency and other day-to-day operations. 21
  • Chief Nursing Officer (CNO): A chief nursing officer works in hospitals, clinics, government organizations and other healthcare organizations to ensure they provide high-quality care to patients. 22 They serve in a leadership capacity, working with other organizational leaders to maintain patient care standards. These nurse leaders establish patient care standards, design patient care systems, integrate medical technologies effectively and collaborate with other healthcare professionals and hospital leaders.
  • Health Policy Analyst: Nurses with DNPs can work in healthcare policy , which impacts everything from healthcare access, cost, delivery methods and privacy. They often work for government or nonprofit organizations, higher education, research laboratories and hospitals to develop healthcare policies based on data and research. 23
  • Nursing informaticist: Nursing informaticists combine nursing with computer and information science to develop medical data systems such as electronic medical records that support nurses and improve patient care outcomes. 24 They may educate others on how to use technology, research and develop software or serve as chief nursing officers.
  • Certified Nurse Educator (CNE): A CNE often continues to practice part-time while teaching undergraduate and graduate nursing courses at a college or university. 25

Benefits of Earning a DNP in Nursing

There are several benefits to earning a DNP in Nursing:

  • Increased salary. Pay for nurses varies based on education level and length of experience, which means that nurses with a DNP will make more than an RN with an MSN. 26 Their advanced education and access to leadership roles may lead to higher earning potential than other nurses.
  • Access to new career opportunities. 8 Nurses with DNPs have access to more leadership roles than nurses who hold an MSN, especially those with the Nurse Executive specialization. These leadership positions mean those with a DNP can help shape the future of healthcare policy .
  • Increased autonomy. 27 Nurses with DNPs who choose to be nurse practitioners may have more autonomy than traditional RNs. They may also leave clinical practice to work as a healthcare leader.
  • Advanced expertise. A DNP is a terminal degree, so completing a DNP program allows you to advance your education and deepen your expertise in a specialized area.

DNP vs PhD in Nursing.

The main differences between a DNP and PhD in Nursing relate to career goals, program length and clinical requirements.

The DNP prepares students for active practice, while a PhD in Nursing prepares students for careers in research and academia. 1 While nurses with a DNP may qualify to teach while they practice, nurses with a PhD will typically not interact with patients outside of a research setting. 28

While it varies by program, entry point, specialization and enrollment type, a PhD in Nursing generally takes longer to complete than a DNP. Most students earn a DNP in two to four years Please note the DNP-FNP program typically takes 3 years and 8 months to complete, based off the most recent 2-year graduate data (2020-2021 and 2021-2022); however, individual experiences will vary based on factors including, but not limited to individual progress, traditional vs. accelerated pathway, credits transferred, and other factors. , while it takes most students four to six years to earn their PhD in Nursing. 1

Because a DNP prepares you for clinical practice, a DNP program will require some clinical experience to prepare for clinical practice. Since nurses with a PhD in Nursing generally work only with researchers, scientists and future nurses rather than patients, the degree does not require students to complete clinical hours. 28

Earn Your DNP with USAHS

A PhD in Nursing and a DNP can help you propel your nursing career forward, but a DNP comes with more career options and a shorter timeline, making it a better option for many nurses. Influence health outcomes by caring for patients, advocating for better healthcare policies or leading organizations with a DNP from USAHS.

If you’re considering whether to earn your PhD vs DNP degree, explore our DNP program or apply today . Enter with a BSN or MSN and pursue the right path for you.

  • Registered Nursing.org, “Getting Your PhD in Nursing,” Registered Nursing.org , https://www.registerednursing.org/phd/ .
  • Gayle Morris, “Bachelor of Science in Nursing (BSN) Overview,” Nurse Journal , last updated October 2023, https://nursejournal.org/degrees/bsn/ .
  • NurseJournal Staff, “How Long Is Nursing School? Breakdown by Degree and Type of Nurse,” Nurse Journal , last updated May 2023, https://nursejournal.org/resources/how-long-is-nursing-school/ .
  • Sarah Kividen, “What Is the NCLEX?”, Nurse.org , last updated May 2023, https://nurse.org/resources/what-is-the-nclex/ .
  • American Association of Colleges of Nursing, “PhD Education,” American Association of Colleges of Nursing , https://www.aacnnursing.org/students/nursing-education-pathways/phd-education .
  • Jennifer Schlette, “10 Pros and Cons of Ph.D. in Nursing,” Nursing Process https://www.nursingprocess.org/phd-nursing-pros-and-cons.html .
  • EveryNurse Staff, “PhD in Nursing,” EveryNurse , last modified January 2022, https://everynurse.org/phd-programs/ .
  • Danielle LeVeck, “What Is a DNP and Is It Worth It?” Nurse.org , last updated September 2023, https://nurse.org/articles/how-to-get-a-dnp-is-it-worth-it/ .
  • Angelina Walker, “RN Licensing requirements by State,” Nurse.org , last updated July 2022, https://nurse.org/articles/guide-to-registered-nurse-licensure-by-state/ .
  • NurseJournal Staff, “Doctor of Nursing Practice (DNP) Degree Overview,” Nurse Journal , last updated September 2023, https://nursejournal.org/degrees/dnp/ .
  • American Association of Colleges of Nursing, “DNP Education,” American Association of Colleges of Nursing , https://www.aacnnursing.org/students/nursing-education-pathways/dnp-education .
  • Ann Feeny, “What Is the Average Salary With a DNP Nurse Journal , last updated November 2022, https://nursejournal.org/degrees/dnp/average-salary-with-a-dnp-degree/ .
  • American Association of Nurse Practitioners, “Are You Considering a Career as an Adult-Gerontology Primary Care Nurse Practitioner?” American Association of Nurse Practitioners , last updated June 2020, https://www.aanp.org/news-feed/are-you-considering-a-career-as-an-adult-gerontology-primary-care-nurse-practitioner .
  • American Association of Nurse Practitioners, “  “Are You Considering a Career as a Neonatal Nurse Practitioner?” American Association of Nurse Practitioners , last updated March 2020, https://www.aanp.org/news-feed/are-you-considering-a-career-as-neonatal-nurse-practitioner .
  • RegisteredNursing.org Staff Writers “Pediatric Nurse Practitioner,” RegisteredNursing.org , last updated June 2023, https://www.registerednursing.org/nurse-practitioner/pediatric/ .
  • Nursing License Map, “How To Become a Pyschiatric Mental Health Nurse Practitioner (PMHNP),” Nursing License Map , last updated October 2021, https://nursinglicensemap.com/advanced-practice-nursing/nurse-practitioner/psychiatric-and-mental-health-nurse-practitioner-pmhnp/ .
  • American Association of Nurse Practitioners, “Are You Considering a Career as a Women’s Health Nurse Practitioner?” American Association of Nurse Practitioners , last updated April 2020, https://www.aanp.org/news-feed/are-you-considering-a-career-as-a-womens-health-nurse-practitioner .
  • Nichole Galan, “Certified Nurse Midwife Career Overview, Nurse Journal , last updated May 2023, https://nursejournal.org/nurse-midwife/ .
  • Cleveland Clinic, “Nurse Anesthetist (CRNA),” Cleveland Clinic , last updated March 2022, https://my.clevelandclinic.org/health/articles/22561-nurse-anesthetist-crna .
  • NurseJournal Staff, “NP vs CNS: What’s the Difference?” Nurse Journal , last updated March 2023, https://nursejournal.org/resources/np-vs-cns/ .
  • American Nurses Association, “Charge Nurse vs. Nurse Manager: What’s the Difference?” American Nurses Association , https://www.nursingworld.org/resources/individual/charge-nurse-vs-nurse-manager/ .
  • NurseJournal Staff, “What Is a Chief Nursing Officer?” Nursing Journal , last updated March 2023, https://nursejournal.org/careers/chief-nursing-officer/ .
  • Carol Dolan, “What Is a Health Policy Analyst?” Master’s in Public Health Degree Programs , https://www.masterspublichealth.net/faq/what-is-a-healthcare-policy-analyst/ .
  • RegisteredNursing.com Staff Writers, “Nursing Informatics,” RegisteredNursing.org , last updated August 2023, https://www.registerednursing.org/nursing-informatics/ .
  • Nursing License Map, “What Is a Doctor of Nursing Practice (DNP) Degree?” Nursing License Map , last updated January 2022, https://nursinglicensemap.com/nursing-degrees/dnp-degree-programs/what-is-a-dnp-degree/ .
  • Registered Nursing.org, “RN Salary — What To Expect,” Registered Nursing.org , https://www.registerednursing.org/rn-salary/ .
  • Kasee Wiesen, “Exploring the Benefits of a DNP: Is It Right for You?” Health eCareers , last updated July 2023, https://www.healthecareers.com/career-resources/nurse-credentialing-and-education/benefits-of-a-dnp-in-nursing .
  • Thrive AP, “DNP vs PhD in Nursing: What’s the Difference?” Thrive AP https://provider.thriveap.com/blog/dnp-vs-phd-nursing-whats-difference .

ARE YOU INSPIRED?

There could be an article about you here one day. Take charge of your own life-story!

Take charge of your own life-story

More Nursing MSN & DNP Articles

phd nursing meaning

Dr. Elisabeth McGee Discusses USAHS’ Driving Role in Future-Focused Healthcare

phd nursing meaning

Nurse Practitioner vs. Physician Assistant: Key Differences

phd nursing meaning

85 Nursing Quotes: Words of Wisdom for Nurses

Upcoming nursing msn & dnp events.

Miami, FL Commencement - Wednesday, August 21, 2024  4:00pm-6:00pm ET | USAHS

Miami, FL Commencement - Wednesday, August 21, 2024 4:00pm-6:00pm ET | USAHS

San Marcos, CA Commencement - Wednesday, August 21  4:00pm-6:00pm PT | USAHS

San Marcos, CA Commencement - Wednesday, August 21 4:00pm-6:00pm PT | USAHS

St. Augustine, First and Post Professional - Commencement - Friday, August 23, 2024  3:00pm-5:00pm ET | USAHS

St. Augustine, First and Post Professional - Commencement - Friday, August 23, 2024 3:00pm-5:00pm ET | USAHS

American Association of Colleges of Nursing - Home

PhD Education

Find Programs

There is no doubt that education is the path for a nurse to achieve greater clinical expertise. At the same time, however, the nursing profession needs more nurses educated at the doctoral level to replenish the supply of faculty and researchers. The national shortage of faculty will soon reach critical proportions, having a significant impact on educational programs and their capacity to educate future generations of nursing students.

Although the number of doctorate programs has continued to increase, the total enrollment of students in these programs has remained fairly constant, resulting in a shortage of newly minted PhDs to renew faculty ranks.  As a result, approximately 50% of nursing faculty possess the doctorate as a terminal degree. Furthermore, with many advances being made in the treatment of chronic illnesses, there is a continuing need for research that assists patients in living with their illness.  This research requires individual investigators who are prepared on the doctoral level.

One reason there is a lack of nurses prepared at the doctoral level is that, compared with other professions, nurses have more interruptions in their careers. Many in the profession are females who work as nurses while fulfilling responsibilities as wives and mothers.  As a result, many pursue their education on a part-time basis. Also, the nursing profession traditionally has viewed clinical experience as being a prerequisite to graduate education. This career path results in fewer individuals completing the doctorate at an earlier stage in their career, thereby truncating their productivity as academics, researchers, and administrators. To reverse this trend, many nursing schools have developed programs that admit students into graduate (doctorate and master's) programs directly from their undergraduate or master's programs.

Nursing Research

When nurses do research for their doctorates, many people tend to think that it focuses primarily on nurses and nursing care. In reality, nurses carry out clinical research in a variety of areas, such as diabetes care, cancer care, and eating disorders. 

In the last thirty years, advances in medicine have involved, for the most part, advancing treatment not cures. In other words, no cure for the illness has been discovered, but treatment for that illness has improved. However, sometimes the treatment itself causes problems for patients, such as the unwelcome side effects of chemotherapy. Nurses have opportunities to devise solutions to problems like these through research, such as studies on how to manage the illness and its treatment, thereby allowing individuals to lead happy and productive lives.

The Curricula

Doctoral programs in nursing are aimed at preparing students for careers in health administration, education, clinical research, and advanced clinical practice. Basically, doctoral programs prepare nurses to be experts within the profession, prepared to assume leadership roles in a variety of academic and clinical settings, course work and research, students are trained as researchers and scholars to tackle complex health-care questions. Program emphasis may vary from a focus on health education to a concentration on policy research. The majority of doctoral programs confer the Doctor of Philosophy (PhD) degree, but some award the Doctor of Nursing Science (DNS), and the Doctor of Education (EdD).

Doctoral nursing programs traditionally offer courses on the history and philosophy of nursing and the development and testing of nursing and other healthcare techniques, as well as the social, economic, political, and ethical issues important to the field. Data management and research methodology are also areas of instruction. Students are expected to work individually on research projects and complete a dissertation.

Doctoral programs allow study on a full- or part-time basis. For graduate students who are employed and therefore seek flexibility in their schedules, many programs offer courses on weekends and in the evenings.

Admission Requirements

Admission requirements for doctoral programs vary. Generally, a master's degree is necessary, but in some schools a master's degree is completed in conjunction with fulfillment of the doctoral degree requirements. Standard requirements include an RN license, Graduate Record Examinations (GRE) scores, college transcripts, letters of recommendation, and an essay. Students applying for doctoral-level study should have a solid foundation in nursing and an interest in research. Programs are usually the equivalent of three to five years of full-time study.

Selecting a Doctoral Program

Selecting a doctoral program comes down to personal choice. Students work closely with professors, and, thus, the support and mentoring you receive while pursuing your degree is as vital as the quality of the facilities. The most important question is whether there is a "match" between your research interest and faculty research. Many of the same questions you would ask about baccalaureate and master's degree programs apply to doctoral programs. However, in a doctoral program, the contact with professors, the use of research equipment and facilities, and the program's flexibility in allowing you to choose your course of study are critical.

Some questions to consider asking include: Are there opportunities to present research findings at professional meetings? Is scholarship of faculty, alumni, and students presented at regional and national nursing meetings and subsequently published? Has the body of research done at a university enhanced the knowledge of nursing and health care?

Other questions to consider include: Does the university consider research a priority? Does the university have adequate funding for student research? Many nurses with doctorate degrees make the natural transition into an academic career, but there are many other career options available for nurses prepared at this level. For example, nurses prepared at the doctoral level are often hired by large consulting firms to work with others in designing solutions to health-care delivery problems. Others are hired by large hospital chains to manage various divisions, and some nurses with doctorate degrees are hired to manage complex healthcare systems at the executive level. On another front, they conduct research and formulate national and international healthcare policy. In short, because of the high level of education and a shortage of nurses prepared at this level, there are a number of options.

Salaries are related to the various positions.  Faculty salaries vary by the type of institution and by faculty rank, typically ranging from approximately $80,000 at the assistant professor level to over $115,000 at the professor level.  Salaries of nurse executives also vary, with the lowest salaries being in small rural hospitals and the highest being in complex university medical centers. Consultant salaries are wide ranging but often consist of a base plus some percentage of work contracted. Clinical and research positions vary considerably by the type of institution and the nature of the work.  Needless to say, a doctoral education does provide individuals with a wide range of opportunities, with salaries commensurate with the type and level of responsibilities. 

IMAGES

  1. DNP vs PhD in Nursing- Which one is best?

    phd nursing meaning

  2. What is Phd in Nursing Full information |Phd Nursing in India| Nursing Career|Nursing Course |

    phd nursing meaning

  3. PhD in Nursing- Eligibility Criteria, Admission, Process, Salary and

    phd nursing meaning

  4. Terminal Degree Nursing Options: PhD vs. DNP

    phd nursing meaning

  5. Career Benefits of PhD in Nursing

    phd nursing meaning

  6. The Nursing PhD and DNP, What's the Difference?

    phd nursing meaning